Gönderen Konu: Eşitsizlik Çalışma Soruları  (Okunma sayısı 6409 defa)

Çevrimdışı MATSEVER 27

  • Geo-Maniac
  • ********
  • İleti: 738
  • Karma: +10/-6
Eşitsizlik Çalışma Soruları
« : Şubat 16, 2016, 08:32:04 ös »
Analiz Cebir Eşitsizlikler Çalışma Kağıdı

$17.02.2016$

$\textit{Problem 1}$

$a+b+c=1$ eşitliğini sağlayan tüm $a,b,c$ pozitif gerçel sayıları için;
$${{a-bc}\over{a+bc}} + {{b-ca}\over{b+ca}} + {{c-ab}\over{c+ab}}
\leq {3 \over 2}$$
olduğunu gösteriniz.



$\textit{Problem 3}$

$abc=1$ eşitliğini sağlayan tüm $a,b,c$ pozitif gerçel sayıları için;
$$\frac{ab}{a^5+b^5+ab}+\frac{bc}{b^5+c^5+bc}+\frac{ca}{a^5+c^5+ca}\leq1$$
olduğunu gösteriniz.



$\textit{Problem 4}$

$a,b,c$ pozitif gerçel sayıları $a^2+b^2+c^2+2abc \le 1$ koşulunu sağlıyorsa;
$$K \left(\dfrac{1}{abc}-\dfrac{a}{b}-\dfrac{b}{c}-\dfrac{c}{a} \right) > 2(a+b+c) \left(\dfrac{1}{2a+1}+\dfrac{1}{2b+1}+\dfrac{1}{2c+1}+2(ab+bc+ca) \right)$$
olmasını sağlayan en küçük $K$ tamsayı sabitini belirleyiniz.



$\textit{Problem 5}$

$a+b+c=3$ koşulunu sağlayan tüm $a,b,c$ pozitif gerçel sayıları için;
$$\dfrac{2-\sqrt{a}}{\sqrt{c+3a}}+\dfrac{2-\sqrt{b}}{\sqrt{a+3b}}+\dfrac{2-\sqrt{c}}{\sqrt{b+3c}} \ge \frac{3}{2}$$
olduğunu gösteriniz.



$\textit{Problem 6}$

$a,b,c $ pozitif gerçel sayıları için $a^2+b^2+c^2+abc \le 4$ koşulu sağlanıyorsa;
$$\dfrac {1}{\sqrt {a}}+\dfrac {1}{\sqrt {b}}+\dfrac {1}{\sqrt {c}} \ge a+b+c $$
olduğunu gösteriniz.



$\textit{Problem 7}$

$xy+yz+zx \ge 3$ koşulunu sağlayan tüm $x,y,z$ pozitif gerçel sayıları için;
$$\dfrac{x}{(z+1)^2} \sqrt{ \left(\dfrac{x}{y}+z \right)(y+zx)} \text{  +  }\dfrac{y}{(x+1)^2} \sqrt{ \left(\dfrac{y}{z}+x \right)(z+xy)} \text{  +  }\dfrac{z}{(y+1)^2} \sqrt{ \left(\dfrac{z}{x}+y \right)(x+yz)} \ge \dfrac{3}{2}$$
olduğunu gösteriniz.



$\textit{Problem 8}$

$a^3+b^3+c^3=a+b+c$ eşitliğini sağlayan tüm $a,b,c$ pozitif gerçel sayıları için;
$$\dfrac{a}{a^2+b^2+c^3}+\dfrac{b}{b^2+c^2+a^3}+\dfrac{c}{c^2+a^2+b^3} \ge abc$$
olduğunu gösteriniz.



$\textit{Problem 9}$

$x+y+z \le 3$ koşulunu sağlayan tüm $x,y,z$ pozitif gerçel sayıları için;
$$\dfrac{x^3}{x^3+3x-1}+\dfrac{y^3}{y^3+3y-1}+\dfrac{z^3}{z^3+3z-1} \le 1$$
olduğunu gösteriniz.
« Son Düzenleme: Mayıs 01, 2016, 11:23:37 öö Gönderen: MATSEVER 27 »
Vatan uğrunda ölen varsa vatandır.

Çevrimdışı LaçinCanAtış

  • G.O Sevecen Üye
  • ****
  • İleti: 56
  • Karma: +3/-3
Ynt: Eşitsizlik Çalışma Soruları
« Yanıtla #1 : Eylül 11, 2016, 04:50:18 öö »
PROBLEM 3-
$$\left\{ { x }^{ 5 }+{ y }^{ 5 }\ge { x }^{ 2 }{ y }^{ 2 }\left( x+y \right) \quad x,y>0\quad için.\\ \sum { \frac { 1 }{ ab\left( a+b+c \right)  }  } =\sum { \frac { c }{ a+b+c } =1 }  \right\} $$
özellikleri kullanılırsa;
$\sum { \frac { ab }{ { a }^{ 5 }+{ b }^{ 5 }+ab }  } \le \sum { \frac { ab }{ { a }^{ 2 }{ b }^{ 2 }\left( a+b \right) +{ a }^{ 2 }{ b }^{ 2 }c }  } \le \frac { ab }{ { a }^{ 2 }{ b }^{ 2 }\left( a+b+c \right)  } =1$Olur.İspat biter.

Çevrimdışı LaçinCanAtış

  • G.O Sevecen Üye
  • ****
  • İleti: 56
  • Karma: +3/-3
Ynt: Eşitsizlik Çalışma Soruları
« Yanıtla #2 : Eylül 11, 2016, 04:57:03 öö »
PROBLEM 1-
$$a+bc=a\left( a+b+c \right) +bc=\left( a+b \right) \left( a+c \right) \\ \sum { \frac { a-bc }{ \left( a+b \right) \left( a+c \right)  } \le \frac { 3 }{ 2 }  } \Rightarrow \sum { \left( a-bc \right) \left( b+c \right) \le \frac { 3 }{ 2 } \left( a+b \right) \left( b+c \right) \left( a+c \right)  } \\ LHS=2\left( \sum { ab }  \right) -\prod { \left( a+b \right)  } +2abc=2\left( \sum { a }  \right) \left( \sum { ab }  \right) -\prod { \left( a+b \right)  } +2abc\\ =\prod { \left( a+b \right)  } +4abc\le \frac { 3 }{ 2 } \prod { a+b } \Longrightarrow \prod { \left( a+b \right)  } \ge 8abc\\ $$

Çevrimdışı ArtOfMathSolving

  • G.O Efsane Üye
  • *******
  • İleti: 422
  • Karma: +5/-8
Ynt: Eşitsizlik Çalışma Soruları
« Yanıtla #3 : Eylül 11, 2016, 06:09:04 ös »
$$\prod_{cyc}  (a+b) \geqslant 8abc $$

İspatın Geri kalanını yazmayı unutmuşsunuz galiba. İspatın bitmesi için Yukarıdaki ifadenin ispatlanması gerekir. Yani, $\begin{align*}\prod_{cyc}(a+b)=\prod_{cyc}(1-a)=(ab+ac+bc)-(a+b+c)+1-abc\Rightarrow ab+ac+bc\geqslant 9abc\end{align*}$ ifadesinin doğruluğunu göstermeniz ispatı bitirecektir.
« Son Düzenleme: Eylül 11, 2016, 06:13:34 ös Gönderen: ArtOfMathSolving »
Sıradan bir matematikçi...

Çevrimdışı LaçinCanAtış

  • G.O Sevecen Üye
  • ****
  • İleti: 56
  • Karma: +3/-3
Ynt: Eşitsizlik Çalışma Soruları
« Yanıtla #4 : Eylül 11, 2016, 08:41:28 ös »
$$\prod_{cyc}  (a+b) \geqslant 8abc $$

İspatın Geri kalanını yazmayı unutmuşsunuz galiba. İspatın bitmesi için Yukarıdaki ifadenin ispatlanması gerekir. Yani, $\begin{align*}\prod_{cyc}(a+b)=\prod_{cyc}(1-a)=(ab+ac+bc)-(a+b+c)+1-abc\Rightarrow ab+ac+bc\geqslant 9abc\end{align*}$ ifadesinin doğruluğunu göstermeniz ispatı bitirecektir.
LaTeX maalesef zahmetli ve çözümlerimi bu yüzden kısa tutmaya çalışıyorum.Vakit dar,bilirsin.

 


Sitemap 1 2 3 4 5 6 7 8 9 10 11 12 13 14 15 16 17 18 19 20 21 22 23 24 25 26 27 28 29 30 31 32 33 34 35 36 37 
SimplePortal 2.3.3 © 2008-2010, SimplePortal